Compactly supported continuous function is uniformly continuous











up vote
5
down vote

favorite
3












Let $f:mathbb R rightarrow mathbb R$ be continuous and compactly supported. How can I prove that $f$ is uniformly continuous ? I was trying to prove it by contradiction but get stuck. My attempt was as follows:



Let $E$ be the compact support of $f$. On $E$ we know that $f$ is uniformly continuous. If we assume that $f$ is not uniformly continuous we know
$$
exists epsilon > 0 forall delta > 0 exists x,y in mathbb R: |x-y|<delta wedge |f(x)-f(y)| geq epsilon
$$
Let $delta_n := frac 1n$ and fix this $epsilon > 0$. Compute a $delta > 0$ s.t. $forall x,y in E:|x-y| < delta rightarrow |f(x)-f(y)| <epsilon$. Let $n geq N$ s.t. $frac 1N < delta$. Then we may assume wlog that $x_n in E$ and $y_n in mathbb R setminus E$ wehere $x_n,y_n$ are the points corresponding with $delta_n$. This gives a sequence of points where $|x_n-y_n| rightarrow 0$ and $x_n in E$ and $y_n in mathbb R setminus E$.



I now want to use somehow the continuitiy of $f$. How can I do this ? Is this approach a good one ? Can it be more simple ?



New idea:
I know that $E$ is compact so $(y_n)_{n=0}^infty$ has a convergent subsequence $(y_{n_j})_{j=0}^infty$ whit limit say $y in E$. Now
$$
|x_{n_j}-y| leq |x_{n_j}-y_{n_j}|+|y_{n_j}+y| rightarrow 0
$$
So I can take $x_{n_j}$ close to $y$ to get $|f(x_{n_j})-f(y)| = |f(y)| <frac epsilon 2$ by the continuity. I can also take $y_{n_j}$ close to $y$ to get $|f(y_{n_j})-f(y)| <frac epsilon 2$. We further have
$$
|f(y_{n_j})| leq |f(y_{n_j})-f(y)| + |f(y)|
$$
s.t.
$$
|f(y)| geq |f(y_{n_j})| - |f(y_{n_j})-f(y)| geq frac epsilon 2
$$
because $|f(y_{n_j})| geq epsilon$ per construction. So we have $|f(y)| leq frac epsilon 2$ and $|f(y)| > frac epsilon 2$ which is a contradiction.





New solution: Let $epsilon > 0$. Let $delta_1$ for the uniform continuity on $E$. Further
$$
forall x in E, exists delta_x > 0 forall y in mathbb{R}: |x-y|< delta_x rightarrow |f(x)-f(y)| < epsilon
$$
Compute an open finite over of $E$
$$
E subseteq bigcup_{i=1}^N Bleft(x_i,frac{delta_{x_i}}2right)
$$
Write $delta_i := delta_{x_i}$. Let $delta_2 := min_{i=1,cdots,N} frac {delta_i} 2$ and $delta := min(delta_1,delta_2)$.



Assume $|x-y|< delta$. If $x,y in E$ or $x,y notin E$ we are done. Otherwise assume $x in E$ and $y notin E$. Then $x in Bleft(x_i,frac{delta_i}{2}right)$ for some $i$. Further
$$
|y-x_i| leq |y-x|+|x-x_i|leq delta_2 + frac{delta_i}2 leq delta_i
$$
thus $y in B(x_i,delta_i)$ which proves the claim.










share|cite|improve this question




























    up vote
    5
    down vote

    favorite
    3












    Let $f:mathbb R rightarrow mathbb R$ be continuous and compactly supported. How can I prove that $f$ is uniformly continuous ? I was trying to prove it by contradiction but get stuck. My attempt was as follows:



    Let $E$ be the compact support of $f$. On $E$ we know that $f$ is uniformly continuous. If we assume that $f$ is not uniformly continuous we know
    $$
    exists epsilon > 0 forall delta > 0 exists x,y in mathbb R: |x-y|<delta wedge |f(x)-f(y)| geq epsilon
    $$
    Let $delta_n := frac 1n$ and fix this $epsilon > 0$. Compute a $delta > 0$ s.t. $forall x,y in E:|x-y| < delta rightarrow |f(x)-f(y)| <epsilon$. Let $n geq N$ s.t. $frac 1N < delta$. Then we may assume wlog that $x_n in E$ and $y_n in mathbb R setminus E$ wehere $x_n,y_n$ are the points corresponding with $delta_n$. This gives a sequence of points where $|x_n-y_n| rightarrow 0$ and $x_n in E$ and $y_n in mathbb R setminus E$.



    I now want to use somehow the continuitiy of $f$. How can I do this ? Is this approach a good one ? Can it be more simple ?



    New idea:
    I know that $E$ is compact so $(y_n)_{n=0}^infty$ has a convergent subsequence $(y_{n_j})_{j=0}^infty$ whit limit say $y in E$. Now
    $$
    |x_{n_j}-y| leq |x_{n_j}-y_{n_j}|+|y_{n_j}+y| rightarrow 0
    $$
    So I can take $x_{n_j}$ close to $y$ to get $|f(x_{n_j})-f(y)| = |f(y)| <frac epsilon 2$ by the continuity. I can also take $y_{n_j}$ close to $y$ to get $|f(y_{n_j})-f(y)| <frac epsilon 2$. We further have
    $$
    |f(y_{n_j})| leq |f(y_{n_j})-f(y)| + |f(y)|
    $$
    s.t.
    $$
    |f(y)| geq |f(y_{n_j})| - |f(y_{n_j})-f(y)| geq frac epsilon 2
    $$
    because $|f(y_{n_j})| geq epsilon$ per construction. So we have $|f(y)| leq frac epsilon 2$ and $|f(y)| > frac epsilon 2$ which is a contradiction.





    New solution: Let $epsilon > 0$. Let $delta_1$ for the uniform continuity on $E$. Further
    $$
    forall x in E, exists delta_x > 0 forall y in mathbb{R}: |x-y|< delta_x rightarrow |f(x)-f(y)| < epsilon
    $$
    Compute an open finite over of $E$
    $$
    E subseteq bigcup_{i=1}^N Bleft(x_i,frac{delta_{x_i}}2right)
    $$
    Write $delta_i := delta_{x_i}$. Let $delta_2 := min_{i=1,cdots,N} frac {delta_i} 2$ and $delta := min(delta_1,delta_2)$.



    Assume $|x-y|< delta$. If $x,y in E$ or $x,y notin E$ we are done. Otherwise assume $x in E$ and $y notin E$. Then $x in Bleft(x_i,frac{delta_i}{2}right)$ for some $i$. Further
    $$
    |y-x_i| leq |y-x|+|x-x_i|leq delta_2 + frac{delta_i}2 leq delta_i
    $$
    thus $y in B(x_i,delta_i)$ which proves the claim.










    share|cite|improve this question


























      up vote
      5
      down vote

      favorite
      3









      up vote
      5
      down vote

      favorite
      3






      3





      Let $f:mathbb R rightarrow mathbb R$ be continuous and compactly supported. How can I prove that $f$ is uniformly continuous ? I was trying to prove it by contradiction but get stuck. My attempt was as follows:



      Let $E$ be the compact support of $f$. On $E$ we know that $f$ is uniformly continuous. If we assume that $f$ is not uniformly continuous we know
      $$
      exists epsilon > 0 forall delta > 0 exists x,y in mathbb R: |x-y|<delta wedge |f(x)-f(y)| geq epsilon
      $$
      Let $delta_n := frac 1n$ and fix this $epsilon > 0$. Compute a $delta > 0$ s.t. $forall x,y in E:|x-y| < delta rightarrow |f(x)-f(y)| <epsilon$. Let $n geq N$ s.t. $frac 1N < delta$. Then we may assume wlog that $x_n in E$ and $y_n in mathbb R setminus E$ wehere $x_n,y_n$ are the points corresponding with $delta_n$. This gives a sequence of points where $|x_n-y_n| rightarrow 0$ and $x_n in E$ and $y_n in mathbb R setminus E$.



      I now want to use somehow the continuitiy of $f$. How can I do this ? Is this approach a good one ? Can it be more simple ?



      New idea:
      I know that $E$ is compact so $(y_n)_{n=0}^infty$ has a convergent subsequence $(y_{n_j})_{j=0}^infty$ whit limit say $y in E$. Now
      $$
      |x_{n_j}-y| leq |x_{n_j}-y_{n_j}|+|y_{n_j}+y| rightarrow 0
      $$
      So I can take $x_{n_j}$ close to $y$ to get $|f(x_{n_j})-f(y)| = |f(y)| <frac epsilon 2$ by the continuity. I can also take $y_{n_j}$ close to $y$ to get $|f(y_{n_j})-f(y)| <frac epsilon 2$. We further have
      $$
      |f(y_{n_j})| leq |f(y_{n_j})-f(y)| + |f(y)|
      $$
      s.t.
      $$
      |f(y)| geq |f(y_{n_j})| - |f(y_{n_j})-f(y)| geq frac epsilon 2
      $$
      because $|f(y_{n_j})| geq epsilon$ per construction. So we have $|f(y)| leq frac epsilon 2$ and $|f(y)| > frac epsilon 2$ which is a contradiction.





      New solution: Let $epsilon > 0$. Let $delta_1$ for the uniform continuity on $E$. Further
      $$
      forall x in E, exists delta_x > 0 forall y in mathbb{R}: |x-y|< delta_x rightarrow |f(x)-f(y)| < epsilon
      $$
      Compute an open finite over of $E$
      $$
      E subseteq bigcup_{i=1}^N Bleft(x_i,frac{delta_{x_i}}2right)
      $$
      Write $delta_i := delta_{x_i}$. Let $delta_2 := min_{i=1,cdots,N} frac {delta_i} 2$ and $delta := min(delta_1,delta_2)$.



      Assume $|x-y|< delta$. If $x,y in E$ or $x,y notin E$ we are done. Otherwise assume $x in E$ and $y notin E$. Then $x in Bleft(x_i,frac{delta_i}{2}right)$ for some $i$. Further
      $$
      |y-x_i| leq |y-x|+|x-x_i|leq delta_2 + frac{delta_i}2 leq delta_i
      $$
      thus $y in B(x_i,delta_i)$ which proves the claim.










      share|cite|improve this question















      Let $f:mathbb R rightarrow mathbb R$ be continuous and compactly supported. How can I prove that $f$ is uniformly continuous ? I was trying to prove it by contradiction but get stuck. My attempt was as follows:



      Let $E$ be the compact support of $f$. On $E$ we know that $f$ is uniformly continuous. If we assume that $f$ is not uniformly continuous we know
      $$
      exists epsilon > 0 forall delta > 0 exists x,y in mathbb R: |x-y|<delta wedge |f(x)-f(y)| geq epsilon
      $$
      Let $delta_n := frac 1n$ and fix this $epsilon > 0$. Compute a $delta > 0$ s.t. $forall x,y in E:|x-y| < delta rightarrow |f(x)-f(y)| <epsilon$. Let $n geq N$ s.t. $frac 1N < delta$. Then we may assume wlog that $x_n in E$ and $y_n in mathbb R setminus E$ wehere $x_n,y_n$ are the points corresponding with $delta_n$. This gives a sequence of points where $|x_n-y_n| rightarrow 0$ and $x_n in E$ and $y_n in mathbb R setminus E$.



      I now want to use somehow the continuitiy of $f$. How can I do this ? Is this approach a good one ? Can it be more simple ?



      New idea:
      I know that $E$ is compact so $(y_n)_{n=0}^infty$ has a convergent subsequence $(y_{n_j})_{j=0}^infty$ whit limit say $y in E$. Now
      $$
      |x_{n_j}-y| leq |x_{n_j}-y_{n_j}|+|y_{n_j}+y| rightarrow 0
      $$
      So I can take $x_{n_j}$ close to $y$ to get $|f(x_{n_j})-f(y)| = |f(y)| <frac epsilon 2$ by the continuity. I can also take $y_{n_j}$ close to $y$ to get $|f(y_{n_j})-f(y)| <frac epsilon 2$. We further have
      $$
      |f(y_{n_j})| leq |f(y_{n_j})-f(y)| + |f(y)|
      $$
      s.t.
      $$
      |f(y)| geq |f(y_{n_j})| - |f(y_{n_j})-f(y)| geq frac epsilon 2
      $$
      because $|f(y_{n_j})| geq epsilon$ per construction. So we have $|f(y)| leq frac epsilon 2$ and $|f(y)| > frac epsilon 2$ which is a contradiction.





      New solution: Let $epsilon > 0$. Let $delta_1$ for the uniform continuity on $E$. Further
      $$
      forall x in E, exists delta_x > 0 forall y in mathbb{R}: |x-y|< delta_x rightarrow |f(x)-f(y)| < epsilon
      $$
      Compute an open finite over of $E$
      $$
      E subseteq bigcup_{i=1}^N Bleft(x_i,frac{delta_{x_i}}2right)
      $$
      Write $delta_i := delta_{x_i}$. Let $delta_2 := min_{i=1,cdots,N} frac {delta_i} 2$ and $delta := min(delta_1,delta_2)$.



      Assume $|x-y|< delta$. If $x,y in E$ or $x,y notin E$ we are done. Otherwise assume $x in E$ and $y notin E$. Then $x in Bleft(x_i,frac{delta_i}{2}right)$ for some $i$. Further
      $$
      |y-x_i| leq |y-x|+|x-x_i|leq delta_2 + frac{delta_i}2 leq delta_i
      $$
      thus $y in B(x_i,delta_i)$ which proves the claim.







      analysis continuity compactness uniform-continuity






      share|cite|improve this question















      share|cite|improve this question













      share|cite|improve this question




      share|cite|improve this question








      edited Nov 21 at 5:19









      Kei

      297




      297










      asked Jul 17 '13 at 12:45









      Epsilon

      3,46332238




      3,46332238






















          2 Answers
          2






          active

          oldest

          votes

















          up vote
          1
          down vote



          accepted










          Let us first see what is continuity. Given a positive $epsilon$, continuity of $f$ means that at each $x$, we can find a small $delta$ such that $x'in (x-delta,x+delta)$ implies $f(x')in (f(x)-epsilon,f(x)+epsilon)$. So around each $x$, we have a small open set that is mapped very close to $f(x)$. Also note that points in the same small open set are mapped to points close to each other, since they are both mapped to points close to $f(x)$. But the problem here is that this radius of the open sets $delta$ depends on the point $x$.



          Now compactness means you can find a finite collection of such small open sets that they cover the support of the function, and this is good enough for us. Because among these finitely many open sets, you can determine the smallest radius, say $r$. If the distance between $x$ and $x'$ are less than $1/10r$, then you know they must lie in the same open set, then $f(x)$ and $f(x')$ must be close. Now this $r$ does not depend on $x$, you have uniform continuity.






          share|cite|improve this answer





















          • How does it follow that functions that vanish at infinity are uniformly continuous?
            – Chris Cave
            Feb 10 '15 at 11:47










          • Actually this can be answered easily if it is true that a sequence of uniformly continuous functions that converge uniformly to a continuous function, then that function necessarily uniformly continuous.
            – Chris Cave
            Feb 10 '15 at 11:52


















          up vote
          0
          down vote













          Let $K$ be the support of $f$. Define
          $$ M: K times [0,1]to {mathbb R}$$
          by letting
          $$M(x,delta) = max_{|y-x|le delta} |f(y)-f(x)|.$$
          Observe
          $$ |M(x,delta)- M(x',delta')| le |M(x,delta) - M(x,delta')| + |M(x,delta')-M(x',delta')|,$$
          and a quick calculation shows that $M$ is continous. Let ${bar M}(delta) = max_{x} M(x,delta)=M(x_delta,delta)$, due to compactness. Note that $delta to{bar M}(delta)$ is nondecreasing. We show it tends to $0$ as $deltasearrow 0$. If $delta_n searrow 0$ we have a subsequence $x_{delta_{n_k}}to x_0in K$ and therefore by continuity, ${bar M}(delta_{n_k}) = M(x_{delta_{n_k}},delta_{n_k}) to M(x_0,0)=0$.






          share|cite|improve this answer





















            Your Answer





            StackExchange.ifUsing("editor", function () {
            return StackExchange.using("mathjaxEditing", function () {
            StackExchange.MarkdownEditor.creationCallbacks.add(function (editor, postfix) {
            StackExchange.mathjaxEditing.prepareWmdForMathJax(editor, postfix, [["$", "$"], ["\\(","\\)"]]);
            });
            });
            }, "mathjax-editing");

            StackExchange.ready(function() {
            var channelOptions = {
            tags: "".split(" "),
            id: "69"
            };
            initTagRenderer("".split(" "), "".split(" "), channelOptions);

            StackExchange.using("externalEditor", function() {
            // Have to fire editor after snippets, if snippets enabled
            if (StackExchange.settings.snippets.snippetsEnabled) {
            StackExchange.using("snippets", function() {
            createEditor();
            });
            }
            else {
            createEditor();
            }
            });

            function createEditor() {
            StackExchange.prepareEditor({
            heartbeatType: 'answer',
            convertImagesToLinks: true,
            noModals: true,
            showLowRepImageUploadWarning: true,
            reputationToPostImages: 10,
            bindNavPrevention: true,
            postfix: "",
            imageUploader: {
            brandingHtml: "Powered by u003ca class="icon-imgur-white" href="https://imgur.com/"u003eu003c/au003e",
            contentPolicyHtml: "User contributions licensed under u003ca href="https://creativecommons.org/licenses/by-sa/3.0/"u003ecc by-sa 3.0 with attribution requiredu003c/au003e u003ca href="https://stackoverflow.com/legal/content-policy"u003e(content policy)u003c/au003e",
            allowUrls: true
            },
            noCode: true, onDemand: true,
            discardSelector: ".discard-answer"
            ,immediatelyShowMarkdownHelp:true
            });


            }
            });














            draft saved

            draft discarded


















            StackExchange.ready(
            function () {
            StackExchange.openid.initPostLogin('.new-post-login', 'https%3a%2f%2fmath.stackexchange.com%2fquestions%2f445735%2fcompactly-supported-continuous-function-is-uniformly-continuous%23new-answer', 'question_page');
            }
            );

            Post as a guest















            Required, but never shown

























            2 Answers
            2






            active

            oldest

            votes








            2 Answers
            2






            active

            oldest

            votes









            active

            oldest

            votes






            active

            oldest

            votes








            up vote
            1
            down vote



            accepted










            Let us first see what is continuity. Given a positive $epsilon$, continuity of $f$ means that at each $x$, we can find a small $delta$ such that $x'in (x-delta,x+delta)$ implies $f(x')in (f(x)-epsilon,f(x)+epsilon)$. So around each $x$, we have a small open set that is mapped very close to $f(x)$. Also note that points in the same small open set are mapped to points close to each other, since they are both mapped to points close to $f(x)$. But the problem here is that this radius of the open sets $delta$ depends on the point $x$.



            Now compactness means you can find a finite collection of such small open sets that they cover the support of the function, and this is good enough for us. Because among these finitely many open sets, you can determine the smallest radius, say $r$. If the distance between $x$ and $x'$ are less than $1/10r$, then you know they must lie in the same open set, then $f(x)$ and $f(x')$ must be close. Now this $r$ does not depend on $x$, you have uniform continuity.






            share|cite|improve this answer





















            • How does it follow that functions that vanish at infinity are uniformly continuous?
              – Chris Cave
              Feb 10 '15 at 11:47










            • Actually this can be answered easily if it is true that a sequence of uniformly continuous functions that converge uniformly to a continuous function, then that function necessarily uniformly continuous.
              – Chris Cave
              Feb 10 '15 at 11:52















            up vote
            1
            down vote



            accepted










            Let us first see what is continuity. Given a positive $epsilon$, continuity of $f$ means that at each $x$, we can find a small $delta$ such that $x'in (x-delta,x+delta)$ implies $f(x')in (f(x)-epsilon,f(x)+epsilon)$. So around each $x$, we have a small open set that is mapped very close to $f(x)$. Also note that points in the same small open set are mapped to points close to each other, since they are both mapped to points close to $f(x)$. But the problem here is that this radius of the open sets $delta$ depends on the point $x$.



            Now compactness means you can find a finite collection of such small open sets that they cover the support of the function, and this is good enough for us. Because among these finitely many open sets, you can determine the smallest radius, say $r$. If the distance between $x$ and $x'$ are less than $1/10r$, then you know they must lie in the same open set, then $f(x)$ and $f(x')$ must be close. Now this $r$ does not depend on $x$, you have uniform continuity.






            share|cite|improve this answer





















            • How does it follow that functions that vanish at infinity are uniformly continuous?
              – Chris Cave
              Feb 10 '15 at 11:47










            • Actually this can be answered easily if it is true that a sequence of uniformly continuous functions that converge uniformly to a continuous function, then that function necessarily uniformly continuous.
              – Chris Cave
              Feb 10 '15 at 11:52













            up vote
            1
            down vote



            accepted







            up vote
            1
            down vote



            accepted






            Let us first see what is continuity. Given a positive $epsilon$, continuity of $f$ means that at each $x$, we can find a small $delta$ such that $x'in (x-delta,x+delta)$ implies $f(x')in (f(x)-epsilon,f(x)+epsilon)$. So around each $x$, we have a small open set that is mapped very close to $f(x)$. Also note that points in the same small open set are mapped to points close to each other, since they are both mapped to points close to $f(x)$. But the problem here is that this radius of the open sets $delta$ depends on the point $x$.



            Now compactness means you can find a finite collection of such small open sets that they cover the support of the function, and this is good enough for us. Because among these finitely many open sets, you can determine the smallest radius, say $r$. If the distance between $x$ and $x'$ are less than $1/10r$, then you know they must lie in the same open set, then $f(x)$ and $f(x')$ must be close. Now this $r$ does not depend on $x$, you have uniform continuity.






            share|cite|improve this answer












            Let us first see what is continuity. Given a positive $epsilon$, continuity of $f$ means that at each $x$, we can find a small $delta$ such that $x'in (x-delta,x+delta)$ implies $f(x')in (f(x)-epsilon,f(x)+epsilon)$. So around each $x$, we have a small open set that is mapped very close to $f(x)$. Also note that points in the same small open set are mapped to points close to each other, since they are both mapped to points close to $f(x)$. But the problem here is that this radius of the open sets $delta$ depends on the point $x$.



            Now compactness means you can find a finite collection of such small open sets that they cover the support of the function, and this is good enough for us. Because among these finitely many open sets, you can determine the smallest radius, say $r$. If the distance between $x$ and $x'$ are less than $1/10r$, then you know they must lie in the same open set, then $f(x)$ and $f(x')$ must be close. Now this $r$ does not depend on $x$, you have uniform continuity.







            share|cite|improve this answer












            share|cite|improve this answer



            share|cite|improve this answer










            answered Jul 17 '13 at 13:19









            Hui Yu

            6,95022569




            6,95022569












            • How does it follow that functions that vanish at infinity are uniformly continuous?
              – Chris Cave
              Feb 10 '15 at 11:47










            • Actually this can be answered easily if it is true that a sequence of uniformly continuous functions that converge uniformly to a continuous function, then that function necessarily uniformly continuous.
              – Chris Cave
              Feb 10 '15 at 11:52


















            • How does it follow that functions that vanish at infinity are uniformly continuous?
              – Chris Cave
              Feb 10 '15 at 11:47










            • Actually this can be answered easily if it is true that a sequence of uniformly continuous functions that converge uniformly to a continuous function, then that function necessarily uniformly continuous.
              – Chris Cave
              Feb 10 '15 at 11:52
















            How does it follow that functions that vanish at infinity are uniformly continuous?
            – Chris Cave
            Feb 10 '15 at 11:47




            How does it follow that functions that vanish at infinity are uniformly continuous?
            – Chris Cave
            Feb 10 '15 at 11:47












            Actually this can be answered easily if it is true that a sequence of uniformly continuous functions that converge uniformly to a continuous function, then that function necessarily uniformly continuous.
            – Chris Cave
            Feb 10 '15 at 11:52




            Actually this can be answered easily if it is true that a sequence of uniformly continuous functions that converge uniformly to a continuous function, then that function necessarily uniformly continuous.
            – Chris Cave
            Feb 10 '15 at 11:52










            up vote
            0
            down vote













            Let $K$ be the support of $f$. Define
            $$ M: K times [0,1]to {mathbb R}$$
            by letting
            $$M(x,delta) = max_{|y-x|le delta} |f(y)-f(x)|.$$
            Observe
            $$ |M(x,delta)- M(x',delta')| le |M(x,delta) - M(x,delta')| + |M(x,delta')-M(x',delta')|,$$
            and a quick calculation shows that $M$ is continous. Let ${bar M}(delta) = max_{x} M(x,delta)=M(x_delta,delta)$, due to compactness. Note that $delta to{bar M}(delta)$ is nondecreasing. We show it tends to $0$ as $deltasearrow 0$. If $delta_n searrow 0$ we have a subsequence $x_{delta_{n_k}}to x_0in K$ and therefore by continuity, ${bar M}(delta_{n_k}) = M(x_{delta_{n_k}},delta_{n_k}) to M(x_0,0)=0$.






            share|cite|improve this answer

























              up vote
              0
              down vote













              Let $K$ be the support of $f$. Define
              $$ M: K times [0,1]to {mathbb R}$$
              by letting
              $$M(x,delta) = max_{|y-x|le delta} |f(y)-f(x)|.$$
              Observe
              $$ |M(x,delta)- M(x',delta')| le |M(x,delta) - M(x,delta')| + |M(x,delta')-M(x',delta')|,$$
              and a quick calculation shows that $M$ is continous. Let ${bar M}(delta) = max_{x} M(x,delta)=M(x_delta,delta)$, due to compactness. Note that $delta to{bar M}(delta)$ is nondecreasing. We show it tends to $0$ as $deltasearrow 0$. If $delta_n searrow 0$ we have a subsequence $x_{delta_{n_k}}to x_0in K$ and therefore by continuity, ${bar M}(delta_{n_k}) = M(x_{delta_{n_k}},delta_{n_k}) to M(x_0,0)=0$.






              share|cite|improve this answer























                up vote
                0
                down vote










                up vote
                0
                down vote









                Let $K$ be the support of $f$. Define
                $$ M: K times [0,1]to {mathbb R}$$
                by letting
                $$M(x,delta) = max_{|y-x|le delta} |f(y)-f(x)|.$$
                Observe
                $$ |M(x,delta)- M(x',delta')| le |M(x,delta) - M(x,delta')| + |M(x,delta')-M(x',delta')|,$$
                and a quick calculation shows that $M$ is continous. Let ${bar M}(delta) = max_{x} M(x,delta)=M(x_delta,delta)$, due to compactness. Note that $delta to{bar M}(delta)$ is nondecreasing. We show it tends to $0$ as $deltasearrow 0$. If $delta_n searrow 0$ we have a subsequence $x_{delta_{n_k}}to x_0in K$ and therefore by continuity, ${bar M}(delta_{n_k}) = M(x_{delta_{n_k}},delta_{n_k}) to M(x_0,0)=0$.






                share|cite|improve this answer












                Let $K$ be the support of $f$. Define
                $$ M: K times [0,1]to {mathbb R}$$
                by letting
                $$M(x,delta) = max_{|y-x|le delta} |f(y)-f(x)|.$$
                Observe
                $$ |M(x,delta)- M(x',delta')| le |M(x,delta) - M(x,delta')| + |M(x,delta')-M(x',delta')|,$$
                and a quick calculation shows that $M$ is continous. Let ${bar M}(delta) = max_{x} M(x,delta)=M(x_delta,delta)$, due to compactness. Note that $delta to{bar M}(delta)$ is nondecreasing. We show it tends to $0$ as $deltasearrow 0$. If $delta_n searrow 0$ we have a subsequence $x_{delta_{n_k}}to x_0in K$ and therefore by continuity, ${bar M}(delta_{n_k}) = M(x_{delta_{n_k}},delta_{n_k}) to M(x_0,0)=0$.







                share|cite|improve this answer












                share|cite|improve this answer



                share|cite|improve this answer










                answered Nov 21 at 5:51









                Fnacool

                4,956511




                4,956511






























                    draft saved

                    draft discarded




















































                    Thanks for contributing an answer to Mathematics Stack Exchange!


                    • Please be sure to answer the question. Provide details and share your research!

                    But avoid



                    • Asking for help, clarification, or responding to other answers.

                    • Making statements based on opinion; back them up with references or personal experience.


                    Use MathJax to format equations. MathJax reference.


                    To learn more, see our tips on writing great answers.





                    Some of your past answers have not been well-received, and you're in danger of being blocked from answering.


                    Please pay close attention to the following guidance:


                    • Please be sure to answer the question. Provide details and share your research!

                    But avoid



                    • Asking for help, clarification, or responding to other answers.

                    • Making statements based on opinion; back them up with references or personal experience.


                    To learn more, see our tips on writing great answers.




                    draft saved


                    draft discarded














                    StackExchange.ready(
                    function () {
                    StackExchange.openid.initPostLogin('.new-post-login', 'https%3a%2f%2fmath.stackexchange.com%2fquestions%2f445735%2fcompactly-supported-continuous-function-is-uniformly-continuous%23new-answer', 'question_page');
                    }
                    );

                    Post as a guest















                    Required, but never shown





















































                    Required, but never shown














                    Required, but never shown












                    Required, but never shown







                    Required, but never shown

































                    Required, but never shown














                    Required, but never shown












                    Required, but never shown







                    Required, but never shown







                    Popular posts from this blog

                    Le Mesnil-Réaume

                    Ida-Boy-Ed-Garten

                    web3.py web3.isConnected() returns false always